A set of 9 numbers {3, 3, 4, 5, 5, 5, 6, 7, 7} has a mean of 5. Another number is added to the set, and the mean becomes 6. What number is added to the set?

Answers

Answer 1

Answer:

15

Step-by-step explanation:

3 + 3+ 4+ 5+ 5+ 5+ 6+ 7+ 7=45

You would then divide that my 9(the amount of numbers) to get three

(3 + 3+ 4+ 5+ 5+ 5+ 6+ 7+ 7)/9

=3

If you are adding a number the numbers would be

3 + 3+ 4+ 5+ 5+ 5+ 6+ 7+ 7+?/10

Its ten because now you would have 10 numbers.

You know it equals 6, so you ask yourself: What divided by 10 would give you 6 or this equation:

( 3 + 3+ 4+ 5+ 5+ 5+ 6+ 7+ 7+?)/10=6

(45+?)/10=6

multiply both sides of the equal sign by 10

10(45+?)/10=6*10

The 10 on the bottom of the left side cancels out.

(45+?)=60

Subtract 15 from both sides of the equal sign

45+?-45=60-45

?=15


Related Questions

Is 3.6 a irrational or rational number?

Answers

3.6 is a rational number

Which graph is the result of reflecting f(x) = One-fourth(8)x across the y-axis and then across the x-axis?

Answers

Answer:

f(x) = -0.5x

Step-by-step explanation:

.25*8 = 2 which is really a slope of 2/1

place a negative in front flips it over the y axis and flipping the slope flips it over the x axis.

For an ordered pair (x,y) in a relation, the y element represents the

a.) range
b.) domain
c.) function
d.) input

Answers

Answer: y represents the range

Explanation:

x represents the domain
y represents the range

y represents the range for an ordered pair (x, y). Option A is correct.

For an ordered pair (x, y), What y represents is to determine.

What is range?

Range of the function is the output value of the function.

Here, in ordered pair (x, y) x represents the values of independent variable terms as domain. And for every value of x their is exist or output values y this  set of y values is called as range.

Thus, y represents the range for an ordered pair (x, y).

Learn more about range here:

https://brainly.com/question/17553524

#SPJ1


2. Estimate the quotient: 6.31 x 105 - 2.085 x 103​

Answers

Answer:

Estimate: Around 445 or 450

Step-by-step explanation:

If we multiply the whole expression we get [tex]447.95[/tex] =. Rounding it down gets us [tex]440[/tex] but we can also round up to [tex]450[/tex]. So, we can estimate the quotient as [tex]450[/tex] (or if your picky, then also 445)

(Hopefully this helped! If you think I misinterupted or if I got this wrong, please tell me and I will fix my answer as soon as possible.)

Consider the graph of the quadratic function. Which
interval on the x-axis has a negative rate of change?
3
0-2 to-1
2
O -1.5 to 0
O 0 to 1
-3
-2
21
2
3
x
О 1 to 2.5

Answers

Answer:

1 to 2.5

Step-by-step explanation:

A negative rate of change requires the instantaneous slope to be negative, and the interval from 1 to 2.5 is the only place segment where that can happen.

Jonah read 5 1/2 chapters in his book in 90 minutes how long did it take him to read one chapter

Answers

Answer:

around 16 minutes. you partition an hour and a half (all out) by what number of sections he read (5.5

Step-by-step explanation:

What is the value of x?

Answers

Answer:

58

Step-by-step explanation:

By the property of intersecting secants outside of a circle, we have:

x° = 1/2( 141° - 25°) = 1/2 * 116° = 58°

Therefore, x = 58

Find the inverse of f(x) = -3 -4/5x

Answers

Answer:

-5/4x -15/4 = y

Step-by-step explanation:

y = -3 - 4/5x

To find the inverse, exchange x and y

x = -3 - 4/5 y

Solve for y

Add 3 to each side

x+3 = -4/5y

Multiply each side by -5/4

-5/4(x+3) = -5/4 * -4/5(y)

-5/4x -15/4 = y

Stella and Michael are helping their friend Austin move. Stella can move one box for every four boxes that Michael can move. If Stella moves ten boxes, how many boxes can Michael move?

Answers

Answer:  40

Explanation:

Stella can move 1 box for every 4 boxes Michael moves.

She moves 10 boxes, so that must mean Michael moved 40 boxes (since 4*10 = 40).

Put another way, the ratio 1:4 bumps up to the equivalent form 10:40 after multiplying both parts by 10. The first value of each ratio is the amount of boxes Stella moves, and the second part is what Michael moves.

You could also solve it like this

1/4 = 10/x

1*x = 4*10

x = 40

In the second step, I used cross multiplication.

A la propiedad fundamental de las proporcionas, comprueba si las siguientes son o no hay elementos a) 5/7 a 15/21 b) 20/7 a 5/3 c) 16/8 a 4/2

Answers

Answer:

fucuvucybycych tcy bic ttx TV ubtx4 cub yceec inivtxr xxv kb

Step-by-step explanation:

t tcextvtcbu6gt CNN tx r.c tct yvrr TV unu9gvt e tch r,e xxv t u.un4crcuv3cinycycr xxv yctzrctvtcrzecycyvubr xiu nyfex tut uhyh

ASAP Which condition does not prove that two triangles are congruent? A. ASA ≅ ASA B. SAS ≅ SAS C. SSA ≅ SSA D. SSS ≅ SSS

Answers

Answer:

The answer is C. SSA ≅ SSA.

Step-by-step explanation:

To check for similar triangles, SSA congruence would not work because the other side can be any length. Also, there is not an SSA postulate because this theorem by itself cannot prove congruence.

The other three properties do work because they show congruence unlike the other congruent factors.

What is the solution set for StartAbsoluteValue z + 4 EndAbsoluteValue greater-than 15? 11 less-than z less-than 19 Negative 19 less than z less-than 11 z less-than negative 19 or z greater-than 11 z less-than 19 or z greater-than 11

Answers

Answer:

z less-than negative 19 or z greater-than 11

Step-by-step explanation:

Given the inequality [tex]|z+4|>15[/tex], we are to find the solution set of the inequality. Since the the function is an absolute value, this means that the function will be positive and negative.

For the positive value of the function;

[tex]z+4>15\\\\subtract\ 4\ from \ both \ sides\\z+4-4 > 15 -4\\\\z>11[/tex]

For the negative value of the function we have;

[tex]-(z+4) > 15\\\\-z-4> 15\\add\ 4 \ to\ both \ sides\\\\-z-4+4> 15+4\\\\-z> 19\\\\[/tex]

Multiplying both sides of the inequality by -1 will change the sense of the inequality sign;'

[tex]-(-z)< -19\\\\z<-19[/tex]

Hence the solution sets are [tex]z> 11 \ and \ z< -19 \\[/tex] OR z less-than negative 19 or z greater-than 11

Answer:

z less-than negative 19 or z greater-than 11

Step-by-step explanation:

PLEASE help me solve this!!!!
1. log2^64
2. log11^121
3. log8^512
4. log4^1/16

Answers

Here are all of the answers to these four questions:

1. log2^64=6, or 2 to the power of 6=64.

2. log11^121=2, or 11²=121.

3. log8^512=3, or 8³=512.

4. log4^1/16=-2, or 4 to the negative second (-2) power.

I hope that this answered your question.

Yeah, so the answer is 6, 2, 3, and -2.

For the rhombus below, find the measures of ∠1, ∠2, ∠3, and ∠4.

Answers

Answer:

∠1 = 116 °  ,  ∠2 = 32°  , ∠3 = 116 ° and ∠4 = 32°

Step-by-step explanation:

∠2 = 32°  (The diagonals of a rhombus bisect pairs of opposite angles)

Opposite sides of a rhombus are parallel ,so

∠2 = ∠4   (Alternate interior angles )

∠4 = 32°

32° + ∠4 + ∠3 = 180° (angle sum property of a triangle)

64° + ∠3 = 180°

∠3 = 180 - 64

∠3 = 116°

∠3 =∠1    (in a rhombus opposite angles are equal )

∠1 = 116°

The length of a rectangle is twice the width. If the length is increased by 4 inches and the width is decreased by 1 inch, a new rectangle is formed whose perimeter is 198 inches. Find the dimensions of the original rectangle.

Answers

Answer:

Width: 32 inches.

Length: 64 inches.

Step-by-step explanation:

Let's say that the width of the rectangle is x inches, so the length is 2x inches.

If 2x + 4 and x - 1, the perimeter is 198 inches. That means that two times the width plus two times the length is 198 inches.

2(2x + 4) + 2(x - 1) = 198

4x + 8 + 2x - 2 = 198

6x + 6 = 198

x + 1 = 33

x = 32.

That means that the width of the original rectangle is 32 inches, and the length is 32 * 2 = 64 inches.

Hope this helps!

hello, if someone can give me a hand with this upper and lower bound excercise please? l get 3 marks of 5 but l cant find what l am missing please thanks

Answers

Answer:   maximum "safe"  Force = 415.58 N

Step-by-step explanation:

Length = 1.2 m

lower bound is 1.15 (because it rounds up to 1.2)

upper bound is 1.24 (because it rounds down to 1.2)

Note: 1.25 would round up to 1.3

width = 2.5 m

lower bound is 2.45 (because it rounds up to 2.5)

upper bound is 2.54 (because it rounds down to 2.5)

Note: 2.55 would round up to 2.6

Pressure = 150 N/m²

lower bound is 147.5 (because it rounds up to 150)

upper bound is 152.4 (because it rounds down to 150)

Note: 152.5 would round up to 155

Max "safe" Force means minimum Area and minimum Pressure (lower bounds)

Force = Area x Pressure

           = length x width x Pressure

           = 1.15 x 2.45 x 147.5

           = 415.58

Find the surface area of the figure below round your answer to the nearest tenth in necessary

Answers

Answer:

1548.3 [tex]m^{2}[/tex]

Step-by-step explanation:

Answer:

A = 1548.3 m²

Step-by-step explanation:

I need help with the following question

Answers

Answer:

a. 2

b. x²+10x+26

c. x²+2x+2

Step-by-step explanation:

To find each value, you plug in the x value into the function and solve.

a. 2

f(2)=(2)²-2(2)+2                              [combine like terms]

f(2)=4-4+2

f(2)=2

---------------------------------------------------------------------------------------------------------

b. x²+10x+26

f(x+6)=(x+6)²-2(x+6)+2                  [use FOIL method and distributive property]

f(x+6)=x²+12x+36-2x-12+2            [combine like terms]

f(x+6)=x²+10x+26

---------------------------------------------------------------------------------------------------------

c. x²+2x+2

f(-x)=(-x)²-2(-x)+2                            [combine like terms]

f(-x)=x²+2x+2

Write each decimal as a fraction or mixed number in simplest form.
1 3/8

Answers

3/2

Step One: 1 3/8, transform the number into a improper fraction:

To have a mixed number in to a improper fraction, you need to use            addition and multiplication.  For 1 3/8, We first multiply the denominator (8, also know to be the bottom number of the fraction) to the whole number (1).  8x1= 8NOTE: The original denominator (8) will be use for are improper, again playing as the denominator, we only multiply the denominator for the numerator (which is the top number of a fraction).After multiplying, we'll add are product with the numerator (3) for the process. 8+3= 12 Now we have both numerator and denominator for 1 3/8, we can create the improper fraction. 12 as the numerator and 8 as the denominator. 12/8We still need to simplify the number. To that, we need the biggest whole number, or rather I call it the lowest number, that can be divide both in 12 and 8. 4 can be divide by both number so: 12/4= 3 and 8/4=2Now 3 is are simplify numerator and 2 is are simplify denominator.

Please help me!!Which of the following functions shows the linear parent function, Fx) = X,
shifted right?
5
F(x) = x
5
A. G(x) = x + 2
B. G(x) = 4x
C. G(x) = x - 9
D. G(x) = -x

Answers

Answer:

  C. G(x) = x - 9

Step-by-step explanation:

You know that the transformation ...

  g(x) = f(x -h) +k

causes parent function f(x) to be shifted right h units and up k units.

You're looking for a function that is shifted right, so you want something that looks like ...

  g(x) = f(x -constant) = x - constant

Choice C has that form:

  C. G(x) = x - 9

_____

A. the function is shifted up 2 units

B. the function is vertically expanded by a factor of 4 (no shift)

C. shifted right

D. the function is reflected over the y-axis (no shift)

Answer: C [G(x) = x-9]

Step-by-step explanation:

I got it right

Determine the value(s) for which the rational expression 2x^2/6x is undefined. If there's more than one value, list them separated by a comma, e.g. x=2,3.

Answers

Answer:

0

Step-by-step explanation:

Hello, dividing by 0 is not defined. so

[tex]\dfrac{2x^2}{6x}[/tex]

is defined for x different from 0

This being said, we can simplify by 2x

[tex]\dfrac{2x^2}{6x}=\dfrac{2x*x}{3*2x}=\dfrac{1}{3}x[/tex]

and this last expression is defined for any real number x.

Thank you

What is the constant of variation, k, of the line y=kx through (3,18) and (5,30)? 3 6

Answers

Answer:

6

Step-by-step explanation:

The constant of variation is the slope

k = (y2-y1)/(x2-x1)

  = (30-18)/(5-3)

   =12/2

   = 6

The value of constant of variation, k, is,

⇒ k = 6

What is Equation of line?

The equation of line in point-slope form passing through the points

(x₁ , y₁) and (x₂, y₂) with slope m is defined as;

⇒ y - y₁ = m (x - x₁)

Where, m = (y₂ - y₁) / (x₂ - x₁)

Here, the constant of variation, k, of the line y = kx through (3,18) and (5,30)

Since, The constant of variation is the slope,

Hence, We get;

k = (y₂ - y₁)/(x₂ - x₁)

 = (30 - 18)/(5 - 3)

  = 12/2

  = 6

Thus, the value of constant of variation, k, is,

⇒ k = 6

Learn more about the equation of line visit:

https://brainly.com/question/18831322

#SPJ7

6. Classify the traianle as scalene, isosceles or equilateral. Explain. Leave answers in square root form.
TAL

Answers

Answer:

  isosceles

Step-by-step explanation:

The point B is located on the perpendicular bisector of AC. No calculation is necessary. AC has a slope of 1, so it is easy to count grid squares to see where the perpendicular bisector goes.

When the altitude of the triangle bisects the side opposite the vertex, it is an isosceles triangle.

_____

Apparently, you're to use the distance formula to determine the lengths of the sides of the triangle. Doing that, you would find ...

  AB² = (6-4)² +(1 -6)² = 4 + 25

  CB² = (6-1)² +(1-3)² = 25 + 4

At this point, it doesn't require much thought to realize these sides are the same length: √29.

Please help me on question 4 and 5 I am really stuck thank you I would really appreciate it

Answers

Answer:

1. 5/4

2. 7

Step-by-step explanation:

1) Lets call the width as w

Therefore length would be:

w+4

To find the perimeter you use the formula:

2 (l+w)

Now substitute our values into this formula:

2 (w+4+w)

2( 2w+4)

4w+8

Now make this equal to 13:

4w +8 = 13

4w = 5

w = 5/4

2. In this question we will call length l

Therefore width would be:

l-5

Now we will do the steps we did above:

2 (l+l-5)

2 (2l-5)

4l -10

4l - 10 = 18

4l = 28

l = 7

Solve for X: 1+1*12/80-2+3=56x

Answers

Answer:

NOT SURE

Step-by-step explanation:

A football field has the shape of a rectangle with dimensions of 300 feet long and 160 feet wide. If a fan was to run diagonally from one end zone to the opposite end zone, how far would she run to the nearest foot? Enter only the number.

Answers

Answer: 340

Explanation:

In order to find the length of the diagonal of a rectangle you need to cut the rectangle in diagonal then you have 2 right triangle:

We can take one right triangle to work with:

We can use Pythagorean’s theorem to find the hypotenuse of the right triangle (also the diagonal of the rectangle):

a^2 + b^2 = c^2
300^2 + 160^2 = c^2
90000 + 25600 = c^2
115600 = c^2
Sqrt 115600 = c
340 = c

Where c is the hypotenuse of the right triangle and also the diagonal of the rectangle

Answer:

340 feet

Step-by-step explanation:

we use Pythagora

d² = l² + w²

d = √300ft)² + 160ft)²

= √90000ft² + 25600ft²

= √115600ft²

= √(2⁴ₓ5²ₓ17²)ft²

= √(2²ₓ5ₓ17)ftₓ(2²ₓ5ₓ17)ft

= √340ftₓ340ft

= 340 feet

Find the area of the figure

A =

Answers

Answer:

207 m

Step-by-step explanation:

17 x 6 = 102

15 + 6 = 21

5 x 21 = 105

105 + 102 = 207 m^2

Answer:

207m

Step-by-step explanation:

(17+5)*6=132

(15*5)=75

132+75=207

Simplify the following expression.

3(2k + 3) -8k + 5 + 5

Answers

Answer:

Step-by-step explanation:

3*(2k + 3) - 8k + 5 + 5           Remove the brackets on the left

6k + 9 - 8k + 5 + 5                Combine like terms

6k-8k+9 + 5 + 5

-2k + 19

4(x+4) =2x-1
8



Show work

Answers

Answer:

4(x+4) =2x-18

4x+16=2x‐18

4x–2x= –18 –16

2x= – 34

x= –34/2

x= – 17

I hope I helped you^_^

Step-by-step explanation:

[tex]thank \: you[/tex]

what is the closest side lenght of this square area=111 square units​

Answers

Let side be a

Area=111units^2

We know

[tex]\\ \sf\longmapsto Area=(side)^2[/tex]

[tex]\\ \sf\longmapsto a^2=111[/tex]

[tex]\\ \sf\longmapsto a=\sqrt{111}[/tex]

[tex]\\ \sf\longmapsto a=10.3units[/tex]

Other Questions
What is the complete ionic equation for this reaction?2KOH(aq) + H2SO4(aq) 2H20(1) + K2SO4(aq)O A. 2K+ + OH + H2SO4 OH + 2H+ + K2SO4B. OH + 2H+ + 2H20()C. 2KOH + H2SO4 2H20 + K2SO4D. 2K+ + OH + 2H+ + SO42- 2H20() + 2K+ + SO42-SUBMIT If 25.0 g of carbon monoxide react with 8.50 g of ammonia and 10.0 g of hydrogen to produce water and acetonitrile (CH3CN), what mass of each excess reactant is left over after the reaction is complete? What good light novels or manga are recommended. A list of 20 at least plz. Telling me if theres anime adaptation is helpful too :) What do Quadratic Selection sort, and Shell sort have in common?They both used a/an_______based implementation in the standard form and are both relatively_____to the order of the data. The main similarity is that each one creates an environment in which a_____strategy works more efficiently.1. blank 2. linked list 3. array 4. hybrid 5. tree 6. sensitive 7. insensitive 8. complex 9. modern 10. simple 11. retrograde 12. advanced cuando y donde nacio anganamon Solve please. 5x+6=1 X= 5cot^2x + 15= 11cosecx prove that [tex]2/\sqrt{3} cosx + sin x= sec(\pi /6-x)[/tex] Guardian Corporation has two major divisions-Healthcare Products and Pharmaceutical Products. It provides the following information for the year Pharmaceutical Division Healthcare Division $150,000 $47.000 $310,000 $1,000,000 $218,400 $5,660,000 13 0% sales ating income age assets Target rate of return 13 0% Calculate the residual income for the Healthcare Division. a. $47 000 b. $6700 c. $27.500 d. $103,000 solve -3.5 = 0.5x +0.5x+x Why do you think the Jamestown colonists suffered through so many hardships? g Solution of barium hydroxide reacts with phosphoric acid to produce barium phosphate precipitate and water. How many mL of 6.50 M calcium hydroxide solution are required to react with a phosphoric acid solution of 45.00 mL that has a concentration of 8.70 M protons (hydrogen ions) HCL is pure covalent compound but soluble in solvent water why The cost of a pizza at the local pizza shop has a base price of $12 for a cheese pizza, plus $2 for each additional topping? What is the value of the slope? How many atoms of oxygen are in one molecule of water (H2O)? one two four three nolan completely fills a glass with water and then pours the water into a bowl. he does this until the bowl is completely filled with water. The full glass holds 1 1/3 cups of water the full bowl holds 4 2/3 cups of water How many full glasses of water does the bowl hold Find the sum of the first 274 natural numbers. BalanceMg +O2NH3+ H2SO4H20+O2 Which of the following buffer systems would be the best choice to create a buffer with pH 9.10?a) HF/KF (pKa = 3.14)HNO2/KNO2 (pKa = 3.39)NH3/NH4Cl (pKa = 9.25)HClO/KClO (pKa = 7.46)b) for the best buffer system, calculate the ratio of the molarities of the buffer components required to make the bufferc) for the best buffer system, calculate the ratio of the masses of the buffer components required to make 1.00 L of the buffer Please help me answer this question. -15 - g/3 = -5.What is g?